Как получить я \ hbar \ гидроразрыва {\ парциальное F (\ vec p)} {\ парциальное p_i}

Википедия указывает, что следующее отношение «легко показать»: [ Икс я , Ф ( п ) ] "=" я Ф ( п ) п я , однако у меня возникли проблемы с его отображением. Я думаю, что я просто испортил многомерное разложение Тейлора (из Ф ( п ) ). Может ли кто-нибудь из вас рассказать мне об этом или связать меня с сайтом, который будет? Спасибо.


Изменить: вот что я получаю (без использования Икс "=" я п что я еще не доказал):

Ф ( п ) "=" Ф ( 0 ) + Дж "=" 1 3 Ф ( 0 ) п Дж п Дж + 1 2 к "=" 1 3 Дж "=" 1 3 2 Ф ( 0 ) п к п Дж п Дж п к +
так
( Икс я Ф ( п ) Ф ( п ) Икс я ) ψ
"=" Икс я [ Ф ( 0 ) ψ я Дж "=" 1 3 Ф ( 0 ) п Дж ( ψ ) Дж + 2 1 2 к "=" 1 3 Дж "=" 1 3 2 Ф ( 0 ) п к п Дж ( ψ ) Дж ( ψ ) к + ]
[ Ф ( 0 ) Икс я ψ я Дж "=" 1 3 Ф ( 0 ) п Дж ( Икс я ψ ) Дж + 2 1 2 к "=" 1 3 Дж "=" 1 3 2 Ф ( 0 ) п к п Дж ( Икс я ψ ) Дж ( Икс я ψ ) к + ]

где

( Икс я ψ ) Дж "=" Икс я Икс Дж ψ + Икс я ψ Икс Дж "=" дельта я Дж ψ + Икс я ψ Икс Дж
и
( Икс я ψ ) Дж ( Икс я ψ ) к "=" ( дельта я Дж ψ + Икс я ψ Икс Дж ) ( дельта я к ψ + Икс я ψ Икс к ) "=" дельта Дж к ψ 2 + Икс Дж ψ ψ Икс к + Икс к ψ Икс Дж ψ + Икс я 2 2 ψ Икс Дж Икс к

Таким образом:

( Икс я Ф ( п ) Ф ( п ) Икс я ) ψ
"=" Икс я [ Ф ( 0 ) ψ я Дж "=" 1 3 Ф ( 0 ) п Дж ψ Икс Дж + 2 1 2 к "=" 1 3 Дж "=" 1 3 2 Ф ( 0 ) п к п Дж ψ Икс Дж ψ Икс к + ]
[ Ф ( 0 ) Икс я ψ я Дж "=" 1 3 Ф ( 0 ) п Дж ( дельта я Дж ψ + Икс я ψ Икс Дж ) + 2 1 2 к "=" 1 3 Дж "=" 1 3 2 Ф ( 0 ) п к п Дж ( дельта Дж к ψ 2 + Икс Дж ψ ψ Икс к + Икс к ψ Икс Дж ψ + Икс я 2 2 ψ Икс Дж Икс к ) + ]

Отсюда не похоже, что все эти условия более высокого порядка будут аннулированы.

Это в основном аналогично этому физике.stackexchange.com/q /87038
@Bubble Это намного проще (для меня), потому что Ф ( Икс ) это просто функция, тогда как Ф ( п ) является оператором и, следовательно, должен быть расширен по Тейлору (я думаю?).
На самом деле в этом случае я даже не уверен, что п я имеет смысл. Потому что п на самом деле это не вектор? Его я .
ф для функции ф является вектором. сам по себе не является вектором, но вам может сойти с рук притвориться, что это так. Когда люди пишут п я "=" я я они действительно имеют в виду п я ф "=" я ( ф ) я , что имеет смысл. Пока ты помнишь это в конечном итоге действует на что-то, и это создает вектор, вы можете сойти с рук, притворяясь, что является вектором.
Привет @Боб Дилан. Комментарии: 1. Обратите внимание, что оператор положения Икс ^ я и оператор импульса п ^ Дж (до знака минус) вступают на паритетных началах в ЦКР , ср. комментарий выше от Bubble. 2. Отметим также, что искомая формула не зависит от операторного представления. Например, можно работать в импульсном представлении, преобразованном Фурье, или можно работать в формализме, явно независимом от представления.
Также по теме: physics.stackexchange.com/q/98372/25301
@RobinEkman полезнее рассматривать операторы положения и импульса как матрицы с элементами по диагонали, особенно когда мы переходим к формализму Дирака.
Я не уверен, почему ты так сказал Ф ( п ) является оператором - это функция импульса, которая может быть эквивалентно представлена ​​​​в виде Ф ( Икс ) . Половина того, чтобы быть действительно хорошим в QM, состоит в том, чтобы менять базис по мере необходимости. В КМ положение и импульс являются преобразованиями Фурье друг друга, что является удивительной симметрией.

Ответы (3)

Как отметил @Qmechanic в комментарии, мы можем использовать любое представление оператора. В импульсном пространстве, Икс ^ "=" + я   / п и п ^ "=" п , так

[ Икс ^ я , Ф ( п ^ ) ] "=" [ я п я , Ф ( п ) ] "=" я ф [ п я , Ф ( п ) ] ф "=" я ф ( п я [ Ф ( п ) ф ] Ф ( п ) ф п я ) "=" я Ф ( п ) п я

Выберите представление импульса,

Икс я "=" я п я

распределять я и подействовать коммутатором на вектор ψ ,

[ Икс я , Ф ( п ) ] ψ "=" я ( п я ( Ф ( п )   ψ ) Ф ( п ) п я ψ )

и применим правило произведения:

"=" я ( Ф ( п ) п я ψ + Ф ( п ) ψ п я Ф ( п ) ψ п я )

"=" я Ф ( п ) п я ψ .

Мы ушли ψ не указано, поэтому:

[ Икс я , Ф ( п ) ] "=" я Ф ( п ) п я

Коммутация двух переменных в некоторых случаях может быть связана со скобкой Пуассона через

[ А ^ , Б ^ ] "=" я { А ^ , Б ^ }
Таким образом,
(1) [ А ^ , Б ^ ] "=" я я ( А д я Б п я А п я Б д я )
Формально А ^ "=" А ( д ^ , п ^ ) и Б ^ "=" Б ( д ^ , п ^ ) . Вы должны быть в состоянии использовать (1) для решения вашей проблемы. Обратите внимание, однако, что это решение не работает во всех случаях (см. Это и это , на которые указал Qmechanic), поскольку это приближение, которое справедливо только в определенных случаях.

Общее решение включает скобку Мойала ,

[ А ^ , Б ^ ] { { А ^ , Б ^ } } А ^ Б ^ Б ^ А ^
где обозначает продукт Moyal star-product (см. ответы либо в этом посте , либо в этом посте, чтобы узнать больше о продукте Moyal). Вышеупомянутое тогда может быть записано как
{ { А ^ , Б ^ } } "=" { А ^ , Б ^ } + О ( 2 )
где { , } вот приведенная выше скобка Пуассона и О ( 2 ) — поправки (называемые деформациями скобки Пуассона).

Таким образом, уравнение (1) становится

(2) [ А ^ , Б ^ ] "=" я я ( А д я Б п я А п я Б д я ) + О ( 2 )

Комментарий к ответу (v1): Метод замены квантовых коммутаторов [ ф ^ , г ^ ] с классическими скобками Пуассона { ф , г } п Б могут пропускать высшие квантовые поправки, ср. например это , это и это посты Phys.SE. На вопрос ОП скобка Пуассона дает точный ответ.